Difference between revisions of "2013 AIME II Problems/Problem 14"

(The Proof)
Line 31: Line 31:
  
 
===The Proof===
 
===The Proof===
The solution presented above does not prove why F(x) is found by dividing x by 3. Indeed, that is the case, as rigorously shown below.
+
The solution presented above does not prove why <math>F(x)</math> is found by dividing <math>x</math> by <math>3</math>. Indeed, that is the case, as rigorously shown below.
  
Consider the case where x = 3k. We shall prove that F(x) = f(x, k+1).
+
Consider the case where <math>x = 3k</math>. We shall prove that <math>F(x) = f(x, k+1)</math>.
For all x/2 >= n > k+1, x = 2n + q, where 0 <= q <= n. This is because x > 3k + 3 = 3n and x < n. Also, as n increases, q decreases. Thus, q = f(x, n) < f(x, k+1) = k - 2 for all n > k+1.
+
For all <math>x/2 >= n > k+1, x = 2n + q</math>, where <math>0 <= q <= n</math>. This is because <math>x > 3k + 3 = 3n</math> and <math>x < n</math>. Also, as n increases, <math>q</math> decreases. Thus, <math>q = f(x, n) < f(x, k+1) = k - 2</math> for all <math>n > k+1</math>.
Consider all n < k+1. f(x, k) = 0 and f(x, k-1) = 3. Also, 0 < f(x, k-2) < k-2. Thus, for k > 5, f(x, k+1) > f(x, n) for n < k+1.
+
Consider all <math>n < k+1. f(x, k) = 0</math> and <math>f(x, k-1) = 3</math>. Also, <math>0 < f(x, k-2) < k-2</math>. Thus, for <math>k > 5, f(x, k+1) > f(x, n)</math> for <math>n < k+1</math>.
  
Similar proofs apply for x = 3k + 1 and x = 3k + 2. The reader should feel free to derive these proofs himself.
+
Similar proofs apply for <math>x = 3k + 1</math> and <math>x = 3k + 2</math>. The reader should feel free to derive these proofs himself.
  
 
==See Also==
 
==See Also==
 
{{AIME box|year=2013|n=II|num-b=13|num-a=15}}
 
{{AIME box|year=2013|n=II|num-b=13|num-a=15}}
 
{{MAA Notice}}
 
{{MAA Notice}}

Revision as of 19:05, 18 October 2014

Problem 14

For positive integers $n$ and $k$, let $f(n, k)$ be the remainder when $n$ is divided by $k$, and for $n > 1$ let $F(n) = \max_{\substack{1\le k\le \frac{n}{2}}} f(n, k)$. Find the remainder when $\sum\limits_{n=20}^{100} F(n)$ is divided by $1000$.

Solution

Easy solution without strict proof

We can find that

$20\equiv 6 \pmod{7}$

$21\equiv 5 \pmod{8}$

$22\equiv 6 \pmod{8}$

$23\equiv 7 \pmod{8}$

$24\equiv 6 \pmod{9}$

$25\equiv 7 \pmod{9}$

$26\equiv 8 \pmod{9}$

Observing these and we can find that the reminders are in groups of three continuous integers, considering this is true, and we get

$99\equiv 31 \pmod{34}$

$100\equiv 32 \pmod{34}$

So the sum is $5+3\times(6+...+31)+32\times 2=1512$, so the answer is $\boxed{512}$.

The Proof

The solution presented above does not prove why $F(x)$ is found by dividing $x$ by $3$. Indeed, that is the case, as rigorously shown below.

Consider the case where $x = 3k$. We shall prove that $F(x) = f(x, k+1)$. For all $x/2 >= n > k+1, x = 2n + q$, where $0 <= q <= n$. This is because $x > 3k + 3 = 3n$ and $x < n$. Also, as n increases, $q$ decreases. Thus, $q = f(x, n) < f(x, k+1) = k - 2$ for all $n > k+1$. Consider all $n < k+1. f(x, k) = 0$ and $f(x, k-1) = 3$. Also, $0 < f(x, k-2) < k-2$. Thus, for $k > 5, f(x, k+1) > f(x, n)$ for $n < k+1$.

Similar proofs apply for $x = 3k + 1$ and $x = 3k + 2$. The reader should feel free to derive these proofs himself.

See Also

2013 AIME II (ProblemsAnswer KeyResources)
Preceded by
Problem 13
Followed by
Problem 15
1 2 3 4 5 6 7 8 9 10 11 12 13 14 15
All AIME Problems and Solutions

The problems on this page are copyrighted by the Mathematical Association of America's American Mathematics Competitions. AMC logo.png